latex

Thứ Sáu, 20 tháng 9, 2013

Bài toán:  Chứng minh rằng với mọi số dương a, b, c thỏa mãn $ a+b+c=3$ thì $$\dfrac{a^2b}{2a+b} + \dfrac{b^2c}{2b+c} + \dfrac{c^2a}{2c+a} \le \dfrac32$$

Lời giải: Ta sẽ chứng minh:
 $$ \dfrac{a^2b}{2a + b} + \dfrac{b^2c}{2b + c} + \dfrac{c^2a}{2c + a} \le 1.$$ Áp dụng bất đẳng thức $Cauchy-Schwarz$: $$\dfrac{{{a}^{2}}b}{2a+b}={{a}^{2}}b.\dfrac{1}{a+a+b}\le {{a}^{2}}b.\dfrac{1}{9}\left( \dfrac{1}{a}+\dfrac{1}{a}+\dfrac{1}{b} \right)=\dfrac{2ab+{{a}^{2}}}{9}.$$ Tương tự ta có: $$\dfrac{b^2c}{2b + c} \le \dfrac{2bc+{{b}^{2}}}{9}\dfrac{c^2a}{2c + a} \le \dfrac{2ca+{{c}^{2}}}{9}$$
Cộng vế theo vế:  $$\dfrac{{{a}^{2}}b}{2a+b}+\dfrac{b^2c}{2b+c}+\dfrac{{{c}^{2}}a}{2c+a}\le \dfrac{{{a}^{2}}+{{b}^{2}}+{{c}^{2}}+2ab+2bc+2ca}{9}=\dfrac{{{\left( a+b+c \right)}^{2}}}{9}=1.$$

Đẳng thức xảy ra khi và chỉ khi $a=b=c=1.\blacksquare $

Chủ Nhật, 8 tháng 9, 2013

VỀ MỘT BÀI TOÁN BẤT ĐẲNG THỨC

VỀ MỘT BÀI TOÁN BẤT ĐẲNG THỨC
1. Lời nói đầu:
Bất  đẳng thức không thuần nhất là một phần quan trọng, hay và tương đối khó  trong bất đẳng thức vì chúng ta không có một phương pháp thực sự “tốt”  nào để giải quyết loạt các bài toán này. Những lời giải cho những bất  đẳng thức dạng này thường mang những ý tưởng khá hay, độc đáo và thường  là những phương pháp không mẫu mực. Ở bài viết này tác giả xin được giới  thiệu đến bạn đọc một bài toán bất đẳng thức không thuần nhất tương đối  đơn giản nhưng lại có nhiều ứng dụng trong việc giải quyết các bất đẳng  thức thuần nhất và không thuần nhất khác, thậm chí là những bất đẳng  thức trong đề thi học sinh giỏi quốc gia và quốc tế.

(Bài viết  được trích ra từ bài viết cùng tên của tác giả đăng trong chuyên đề Toán  học số 9 của trường Phổ Thông Năng Khiếu Đại học Quốc gia Tp.HCM và  được tác giả bổ sung, điều chỉnh.)

2. Nội dung:
Bài toán gốc: Cho $a, b, c$ là các số thực dương thay đổi bất kì. Chứng minh rằng:
$$a^2+b^2+c^2+2abc+1\geq 2(ab+bc+ca) \ \ \  (1)$$
[I]Lời giải 1: [/I]
Ta sẽ sử dụng phương pháp tam thức bậc hai để chứng minh bài toán. Bất đẳng thức được chuyển về dạng tam thức bậc hai như sau:
$$f(a)=a^2+2a(bc-b-c)+(b-c)^2+1\geq 0$$
* Nếu $bc\geq b+c$ thì ta có ngay điều phải chứng minh.
*  Xét trường hợp ngược lại $bc\leq b+c$, và điều này tương đương với  $(b-1)(c-1)\leq 1$. Khi đó ta tính được biệt thức $\Delta '$ của $f(a)$  là:
$$\Delta '=(bc-b-c)^2-(b-c)^2-1=bc(b-2)(c-2)-1$$
Ta xét hai trường hợp:
Trường hợp 1:Có đúng một trong hai số $b, c$ lớn hơn $2$, số còn lại không lớn hơn  $2$. Trong trường hợp này ta có $(b-2)(c-2)\leq 0$ từ đó suy ra $\Delta  '\leq 0$.
Trường hợp 2: Cả hai số $b, c$ đều không lớn hơn 2. Khi đó theo bất đẳng thức AM-GM, ta có :
$$\Delta '=bc(2-b)(2-c)-1\leq \left[\frac{b+c+(2-b)+(2-c)}{4}\right]^4-1= 0.$$

Tóm  lại trong mọi trường hợp ta đều có $\Delta '\leq 0$. Tức $f(a)\geq 0$  và đây là điều phải chứng minh. Đẳng thức xảy ra khi và chỉ khi  $a=b=c=1.$

Lời giải 2: Theo nguyên lý Dirichlet, ta thấy rằng trong ba số $a, b, c$ sẽ có hai số hoặc cùng $\geq 1$ hoặc cùng $\leq 1$. Giả sử hai số đó là $a, b$ khi đó:
$$(a-1)(b-1)\geq 0.$$
Từ đây, bằng cách sử dụng hằng đẳng thức:
$$a^2+b^2+c^2+2abc+1-2(ab+bc+ca)=(a-b)^2+(c-1)^2+2c(a-1)(b-1)\geq 0$$
Ta thu được ngay bất đẳng thức (1), phép chứng minh hoàn tất.

Lời giải 3: Ta sẽ sử dụng phương pháp dồn biến để chứng minh bài toán. Giả sử $c$ là số bé nhất và đặt:
$$f(a,b,c)=a^2+b^2+c^2+2abc+1-2(ab+bc+ca)$$
Ta có:
$$f(a,b,c)-f(\sqrt{ab},\sqrt{ab},c)=(\sqrt{a}-\sqrt{b})^2(a+b+2\sqrt{ab}-2c)\geq 0$$
Do đó $f(a,b,c)\geq f(\sqrt{ab},\sqrt{ab},c)$, vậy ta chỉ cần chứng minh $f(\sqrt{ab},\sqrt{ab},c)\geq 0$.
Thật vậy, nếu đặt $t=\sqrt{ab}$ thì ta có:
$$f(t,t,c)=2t^2+c^2+2t^2c-2(t^2+2tc)+1=(c-1)^2+2c(t-1)^2\geq 0$$
Bài toán được chứng minh xong.

Lời giải 4: Sử dụng lần lượt [I]bất đẳng thức AM-GM[/I], ta có:
$$2abc+1=abc+abc+1\geq 3\sqrt[3]{a^2b^2c^2}\geq \frac{9abc}{a+b+c}$$
Do đó, ta chỉ cần chứng minh:
$$a^2+b^2+c^2+\frac{9abc}{a+b+c}\geq 2(ab+bc+ca)$$
Thực hiện phép khi triển trực tiếp, ta có bất đẳng thức tương đương với:
$$a^3+b^3+c^3+3abc\geq ab(a+b)+bc(b+c)+ca(c+a)$$
Đúng vì đây chính là bất đẳng thức Schur dạng bậc ba nên ta có điều phải chứng minh.

Bất đẳng thức $(1)$ được Darij Grinberg đề xuất trên diễn đàn toán học Mathlinks.ro vào năm 2004. Mặc dù chỉ là một kết quả đơn giản nhưng bất đẳng thức  này lại có nhiều ứng dụng vào việc chứng minh các bất đẳng thức ba biến.  Sau đây, chúng ta sẽ đi vào xét các bài toán cụ thể để hiểu rõ vì sao  chúng tôi lại nói như vậy.

Bài toán 1.1. (Moscow Mathematical Olympiad 2000). Cho $a, b, c$ là các số dương thỏa mãn $abc = 1$. Chứng minh bất đẳng thức sau
$$a^2 + b^2+c^2+a+b+c \geq 2(ab + bc + ca) \ \ \ (1.1.1)$$      

Lời Giải.
Sử dụng bất đẳng thức AM-GM, ta có
$$a + b + c \geq 3\sqrt[3]{abc} = 3 = 2abc + 1$$.
Vì thế để chứng minh bài toán, ta chỉ cần chứng minh
$$a^2+b^2+c^2+2abc+1 \geq 2(ab+bc+ca)$$
Đây chính là bất đẳng thức (1) nên ta có ngay điều phải chứng minh.
Đẳng thức xảy ra khi và chỉ khi $a = b = c = 1.$

[B]Bài toán 1.2. (Viet Nam Mathematical Olympiad 2006).[/B] Tìm hằng số $k$ lớn nhất để bất đẳng thức
$$\frac{1}{a^2}+\frac{1}{b^2}+\frac{1}{c^2}+3k \geq (k+1)(a+b+c)$$
luôn đúng với mọi $a, b, c$ dương thỏa mãn $abc = 1.$

Lời Giải.
Cho  $a = b = t (t > 0, t = 1)$ và $c =\frac{1}{t^2}$, khi đó $a, b, c$  là các số dương và $abc = 1$. Do đó, theo yêu cầu của bài toán ta phải  có
$$\frac{2}{t^2}+t^4+3k \geq (k+1)(2t+\frac{1}{t^2})$$
Bất đẳng thức này tương đương với mỗi bất đẳng thức trong dãy các bất đẳng thức sau đây
$$\frac{2}{t^2}+t^4-3 \geq (k+1)(2t+\frac{1}{t^2}-3),$$
$$\frac{t^6-3t^2+2}{t^2} \geq \frac{(k+1)(2t^3-3t^2+1)}{t^2}$$
$$\frac{t^2-1)^2(t^2+2)}{t^2} \geq \frac{(k+1)(t-1)^2(2t+1)}{t^2}$$
$$\frac{(t+1)^2(t^2+2)}{2t+1} \geq k+1, \forall t>0$$
Cho $t \to 0^+$, ta được $2 \geq k+1$, suy ra $k \leq 1$. Ta sẽ chứng minh rằng 1 chính là hằng số cần tìm, tức là
$$\frac{1}{a^2}+\frac{1}{b^2}+\frac{1}{c^2}+3 \geq 2(a+b+c).$$
Đặt $x = \frac{1}{a}, y= \frac{1}{b}, z=\frac{1}{c}$ thì ta có $xyz=1, 3=2xyz+1$ và
$$x^2+y^2+z^2+2xyz+1 \geq 2(xy+yz+zx)$$
hiển nhiên đúng theo (1). Vậy ta có kết luận $k_{max}=1.$

Bài toán 1.3. (Mircea Lascu, Romania Junior Team Selecsion Test 2005).
Cho các số dương $ a, b, c$ thỏa mãn điều kiện $(a + b)(b + c)(c + a) = 1.$ Chứng minh rằng
$$ab+bc+ca \leq \frac{3}{4}.$$
Lời Giải
Đặt $x = a + b, y = b + c, z = c + a$ thì ta có $xyz = 1$ và
$$a=\frac{z + x -y }{2}, b=\frac{x+y-z}{2}, c=\frac{y+z-x}{2}.$$
Bất đẳng thức cần chứng minh được viết lại như sau
$$\frac{z + x -y }{2}.\frac{x+y-z }{2}+\frac{x+y-z }{2}.\frac{y+z-x }{2}+\frac{y+z-x }{2}.\frac{z + x -y }{2} \leq \frac{3}{4}$$
Sau khi thu gọn, ta được
$$x^2+y^2+z^2+3 \geq 2(xy+yz+zx),$$
hay là
$$x^2+y^2+z^2+2xyz+1 \geq 2(xy+yz+zx).$$
Đây chính là bất đẳng thức (1) nên ta có điều phải chứng minh.
Đẳng thức xảy ra khi và chỉ khi $a = b = c = \frac{1}{2}$

Bài toán 1.4. (Gabriel Dospinescu, Marian Tetiva, Mircea Lascu).
Chứng minh rằng với mọi số thực dương a, b, c, ta đều có
$$a^2+b^2+c^2 + 2abc + 3 \geq  (a + 1)(b + 1)(c + 1).$$

Lời Giải.
Sau khi khai triển và rút gọn, ta có bất đẳng thức tương đương
$$a^2+b^2+c^2+ abc + 2 \geq ab + bc + ca + a + b + c,$$
hay là
$$2(a^2+b^2+c^2) + 2abc + 4 \geq 2(ab + bc + ca + a + b + c).$$
Theo bất đẳng thức (1), ta có
$$a^2+b^2+c^2 + 2abc + 1 \geq 2(ab + bc + ca).$$
Sử dụng đánh giá này, ta đưa được bài toán về chứng minh
$$a^2+b^2+c^2+2abc+1 \geq 2(ab+bc+ca).$$
Bất đẳng thức này tương đương với bất đẳng thức hiển nhiên đúng là
$$(a-1)^2+(b-1)^2+(c-1)^2 \geq 0$$
Bài toán được chứng minh xong. Đẳng thức xảy ra khi và chỉ khi $a = b = c = 1.$

Bài toán 1.5. (Asian Pacific Mathematical Olympiad 2004).
Chứng minh bất đẳng thức sau luôn đúng với mọi số thực dương a, b, c bất kỳ
$$(a^2 + 2)(b^2+ 2)(c^2+ 2) \geq 9(ab + bc + ca), \ \ (1.5.1)$$

Lời Giải. Bất đẳng thức cần chứng minh tương đương với
$$a^2b^2c^2+3(a^2+b^2+c^2)+2(a^2b^2+b^2c^2+c^2a^2+3)+2 \geq 9(ab+bc+ca).$$
Theo [I]bất đẳng thức AM-GM[/I], ta có
$$3(a^2+b^2+c^2)=3(\frac{a^2+b^2}{2}+\frac{b^2+c^2}{2}+\frac{c^2+a^2}{2}) \geq 3(ab+bc+ca)$$
Và $$2(a^2b^2+b^2c^2+c^2a^2+3)=2 \begin{bmatrix} (a^2b^2+1)+(b^2c^2+1)+(c^2a^2+1) \end{bmatrix} \geq 4(ab+bc+ca).$$
Từ đó, bài toán được đưa về chứng minh
$$a^2+b^2+c^2+a^2b^2c^2+2 \geq 2(ab+bc+ca)$$
Bất đẳng thức này được viết lại thành
$$ \begin{bmatrix} a^2+b^2+c^2+2abc+1-2(ab+bc+c+a) \end{bmatrix}+(abc-1)^2 \geq 0$$
hiển nhiên đugns theo (1). Đẳng thức xảy ra khi và chỉ khi $a=b=c=1.$

NHẬN XÉT: Bài toán còn đúng trong trường hợp $a,b,c$ là các số thực bát kì. Thật vậy, từ chứng mnih trên ta thấy rằng
$$(a^2+2)(b^2+2)(c^2+2)=(  \begin{vmatrix}a \end{vmatrix}^2+2)( \begin{vmatrix}b  \end{vmatrix}^2+2)( \begin{vmatrix}c \end{vmatrix}^2+2) \geq  9(\begin{vmatrix}a \end{vmatrix} \begin{vmatrix}b \end{vmatrix}+  \begin{vmatrix}b \end{vmatrix} \begin{vmatrix}c \end{vmatrix}+  \begin{vmatrix}c \end{vmatrix} \begin{vmatrix}a \end{vmatrix}) \geq  9(ab+bc+ca).$$

Bài toán 1.6. (Crux Mathematicorum).
Chứng minh bất đẳng thức sau luôn đúng với mọi số thực dương $a, b, c$ bất kỳ
$$(a^2+2)(b^2+2)(c^2+2) \geq 3(a+b+c)^2$$

Lời Giải. Tương tự như trên, ta cũng sử dụng phép khai triển trực tiếp và viết lại bất đẳng thức dưới dạng
$$a^2b^2c^2+a^2+b^2+c^2+2(a^2b^2+b^2c^2+c^2a^2+3)+2 \geq 6(ab+bc+ca).$$
Đến đây, ta cũng sử dụng bất đẳng thức AM-GM để thu được
$$2(a^2b^2+b^2c^2+c^2a^2+3) \geq 4(ab+bc+ca),$$
và từ đó đưa được bất đẳng thức về chứng minh
$$a^2+b^2+c^2+a^2b^2c^2+2 \geq 2(ab+bc+ca).$$
Đây  chính là bất đẳng thức (1.5.1) đã được chứng minh ở phần trên. Bài toán  được chứng minh xong. Đẳng thức xảy ra khi $a = b = c = 1.$

[B]Nhận Xét[/B]. Vì $(a+b+c)^2 \geq 3(ab+bc+ca),$ nên từ bài toán này ta có thể dễ dàng suy ra
$$(a^2+2)(b^2+2)(c^2+2) \geq 9(ab+bc+ca)$$
Vậy bài toán này chính là một kết quả mạnh hơn của bất đẳng thức Asian Pacific Mathematical Olypiad 2004. Ngoài ra ta còn có thể làm chặt bất đẳng thức này hơn nữa, ta cùng xét bài toán sau đây.

Bài toán 1.7. (Nguyễn Đình Thi) Cho $a, b, c$ là các số thực dương. Chứng minh rằng
$$(a^2+2)(b^2+2)(c^2+2) \geq 3(a+b+c)^2+(abc-1)^2.$$
.
Lời Giải. Sau khi khai triển và rút gọn, ta được bất đẳng thức tương đương
$$2(a^2b^2+b^2c^2+c^2a^2+3)+a^2+b^2+c^2+2abc+1 \geq 6(ab+bc+ca).$$
Theo [I]bất đẳng thức AM-GM[/I] thì
$$2(a^2b^2+b^2c^2+c^2a^2+3) \geq 4(ab+bc+ca).$$
Do đó ta chỉ cần chứng minh
$$a^2+b^2+c^2+2abc+1 \geq 2(ab+bc+ca).$$
Đây chính là bất đẳng thức (1).
Bài toán được chứng minh xong. Đẳng thức xảy ra khi và chỉ khi $a = b = c = 1.$

Bài toán 1.8. (Nguyễn Văn Huyện). Với mọi số thực dương $a, b, c$ và $k \geq 2$ là một số thực bất kỳ, khi đó ta luôn có bất đẳng thức
$$(a^2+k)(b^2+k)(c^2+k) \geq \frac{(k+1)^2}{3}(a+b+c)^2+k^3-3k-2, \ \ (1.8.1)$$

Lời Giải. Ta cũng thực hiện phép khai triển và viết bất đẳng thức trên thành
$$a^2b^2c^2+k(a^2b^2+b^2c^2+c^2a^2)+k^2(a^2+b^2+c^2)+3k+2 \geq \frac{(k+1)^2}{3}(a+b+c)^2.$$
Sử dụng bất đẳng thức (1.5.1), ta được
$$a^2+b^2+c^2+(a^2+b^2+c^2+a^2b^2c^+2) \geq a^2+b^2+c^2+2(ab+bc+ca)=(a+b+c)^2,$$
từ đó chỉ cần chứng minh được
$$k(a^2b^2+b^2c^2+c^2a^2)+(k^2-2)(a^2+b^2+c^2)+3k \geq \frac{k^2+2k+2}{3}(a+b+c)^2.$$
Theo bất đẳng thức AM-GM, thì
$$(a^2b^2+1)+(b^2c^2+1)+(c^2a^2+1) \geq 2(ab+bc+ca)$$
Suy ra
$$k(a^2b^2+b^2c^2+c^2a^2)+(k^2-2)(a^2+b^2+c^2)+3k \geq 2k(ab+bc+ca)+(k^2-2)(a^2+b^2+c^2)$$
$$=(k^2-k-2)(a^2+b^2+c^2)+k(a+b+c)^2$$
$$ \geq \frac{k^2-k-2}{3}(a+b+c)^2+k(a+b+c)^2$$
$$=\frac{k^2+2k-2}{3}(a+b+c)^2.$$
Bài toán được chứng minh xong. Đẳng thức xảy ra khi và chỉ khi $a = b = c = 1.$

Nhận Xét. Trong (1.8.1) nếu cho $k = 2$, thì ta thu được (1.6.1

Thứ Sáu, 6 tháng 9, 2013

Bài toán:  Cho $a,b,c$ là các số thực dương thỏa $a+b+c=3$. Chứng minh rằng : $\left ( a^{3}+b^{3}+c^{3} \right )\left ( a^{2}-b^{2} \right )\left ( b^{2}-c^{2} \right )\left ( c^{2}-a^{2} \right )\leq \dfrac{729\sqrt{3}}{8}$


Giải: Ta có: $(a^3+b^3+c^3)(a+b)(b+c)(c+a)\leq\dfrac{\left[a^3+b^3+c^3+3(a+b)(b+c)(c+a)\right]^2}{12}=\dfrac{(a+b+c)^6}{12}$
Lại có:$(a-b)^2(b-c)^2(c-a)^2\leq b^2c^2(b-c)^2\leq \dfrac{(b+c)^6}{108}\leq \dfrac{(a+b+c)^6}{108}$
(giả sử $a\leq b \leq c$)
Suy ra : $P\leq \dfrac{(a+b+c)^9}{12\sqrt{108}}$
Đẳng thức xảy ra khi $a=0, b=\dfrac{3-\sqrt{3}}{2}, c=\dfrac{3+\sqrt{3}}{2}$


Bài toán:   Tìm các số nguyên x,y thoả mãn:
$$10x^2+20y^2+24xy+8x-24y+51<0$$

Lời giải:   $$\begin{aligned} pt & \iff 100x^2+200y^2+240xy+80x-240y+510<0 \\ & \iff (10x)^2+ 2 \cdot (10x) \cdot (12y+4)+ (144y^2+96y+16)+56y^2-336y++494<0 \\ & \iff (10x+12y+4)^2+56(y-3)^2-10<0 \\ & \iff (10x+12y+4)^2+56(y-3)^2<10 \end{aligned}$$
Dễ thấy $56(y-3)^2 \ \vdots 56$ với mọi $y \in \mathbb{Z}$. Do đó $y=3$. Nên $(10x+40)^2<10 \iff 10(x+4)^2<1$. Hiển nhiên $x=-4$. 
Vậy $$\boxed{x=-4,y=3}.$$

Thứ Năm, 5 tháng 9, 2013


Bài toán:   Cho $a,b,c>0$. CMR:
$$\dfrac{a^{2}+b^{2}}{a+b}+\dfrac{b^{2}+c^{2}}{b+c}+\dfrac{c^{2}+a^{2}}{c+a}\leq 3\left(\dfrac{a^{2}+b^{2}+c^{2}}{a+b+c}\right)$$

Giải:
$$BDT\iff \dfrac{a^{2}+b^{2}}{a+b}+\dfrac{b^{2}+c^{2}}{b+c}+\dfrac{c^{2}+a^{2}}{c+a}\leq 3(\dfrac{a^{2}+b^{2}+c^{2}}{a+b+c}) \\ \iff (a+b+c)\left[\dfrac{a^{2}+b^{2}}{a+b}+\dfrac{b^{2}+c^{2}}{b+c}+\dfrac{c^{2}+a^{2}}{c+a}\right]\leq 3(a^{2}+b^{2}+c^{2}) \\ \iff \dfrac{c(a^{2}+b^{2})}{a+b}+\dfrac{a(b^{2}+c^{2})}{b+c}+\dfrac{b(c^{2}+a^{2})}{c+a}\leq a^{2}+b^{2}+c^{2} \\ \iff {a^2} + {b^2} + {c^2} - \dfrac{{c\left( {{a^2} + {b^2}} \right)}}{{a + b}} - \dfrac{{a\left( {{b^2} + {c^2}} \right)}}{{b + c}} - \dfrac{{b\left( {{c^2} + {a^2}} \right)}}{{a + c}} \ge 0  \\ \iff \dfrac{{ab{{\left( {a - b} \right)}^2}}}{{\left( {a + c} \right)\left( {b + c} \right)}} + \dfrac{{bc{{\left( {b - c} \right)}^2}}}{{\left( {b + a} \right)\left( {c + a} \right)}} + \dfrac{{ca{{\left( {c - a} \right)}^2}}}{{\left( {c + b} \right)\left( {a + b} \right)}} \ge 0$$
Vì $a,b,c>0$ nên ta được điều phải chứng minh
Dấu = xảy ra $ \Leftrightarrow a = b = c.\blacksquare$
Bài toán:  Chứng minh rằng với mọi a;b;c dương thỏa mãn $ab+bc+ca=3$ thì:
$$2(x^3+y^3+z^3) \geq 6xyz+9.\min \left\{(x-y)^2;(y-z)^2;(z-x)^2\right\}$$

Giải:  Ta sẽ chứng minh BĐT mạnh hơn là:
$$2(x^3+y^3+z^3)\ge 6xyz+3\left[(x-y)^2+(y-z)^2+(z-x)^2\right]\\ \Leftrightarrow x^3+y^3+z^3\ge 3xyz+3(x^2+y^2+z^2-xy-yz-zx)\ (*)$$
Thật vậy: Đặt $p=x+y+z;q=xy+yz+zx=3;r=xyz$ thì:
$$(*)\Leftrightarrow p^3-3pq+3r\ge 3r+3(p^2-3q)\\ \Leftrightarrow p^3-9p-3p^2+27\ge 0\\ \Leftrightarrow (p+3)(p-3)^2\ge 0$$
Hiển nhiên đúng. Dấu bằng xảy ra khi $p=q=3\Leftrightarrow x=y=z=1$
Vậy BĐT ban đầu được c/m hoàn toàn. $\square$

Thứ Ba, 3 tháng 9, 2013

Bài toán: Khẳng định hay phủ định bât đẳng thức sau .
Cho $a,b,c,d>0$ . Chứng minh rằng $$ \frac{a}{2b+3c+5d}+\frac{b}{2c+3d+5a}+\frac{c}{2d+3a+5b}+\frac{d}{2a+3b+5c}\geq \frac{2}{5} $$
Giải:   Sử dụng bất đẳng thức $Cauchy-Schwarz$, ta có
 $$ \begin{aligned} \ & \dfrac{a}{2b+3c+5d}+\frac{b}{2c+3d+5a}+\frac{c}{2d+3a+5b}+\frac{d}{2a+3b+5c} \\ & \geq \dfrac{(a+b+c+d)^2}{7(ab+bc+cd+da)+6(ac+bd)} \\ & =\dfrac{2}{5}+\dfrac{2(a-b)^2+2(b-c)^2+2(c-d)^2+2(d-a)^2+(a-c)^2+(b-d)^2}{5\left[7(ab+bc+cd+da)+6(ac+bd)\right]} \geq \frac{2}{5}. \end{aligned}$$
Đẳng thức xảy ra khi và chỉ khi $a=b=c=d.$
Các phương háp chứng minh bất đẳng thức 
1.Phương pháp dồn biến và định lí dồn biến mạnh

1.1.Phương pháp dồn biến đối với các bất đẳng thức 3 biến 

Ví dụ:  Chứng minh rằng với a$,b,c>0 \ and \  a+b+c=3$ thì:
$2(a^2b^2+b^2c^2+c^2a^2)+3 \leq 3(a^2+b^2+c^2).$

Giải:  Không mất tính tổng quát của bài toán giả sử $a \geq b \geq c$ .
Đặt $t=\dfrac{a+b}{2},u=\dfrac{a-b}{2}$ ta có $a=t+u,b=t-u$
Vì $ab=t^2-u^2 \geq c^2$ nên $2t^2-2c^2-u^2 \geq 0$.
Do đó 
$a^2b^2+b^2c^2+c^2a^2=c^2(a^2+b^2)+a^2b^2 \\ =c^2(2t^2+2u^2)+(t^2-u^2)^2 \\ =-u^2(2t^2-2c^2-u^2)+t^4+2c^2t^2 \leq t^4+2c^2t^2.$
Mặt khác hiển nhiên $a^2+b^2+c^2 \geq 2t^2+c^2$
Thay $c=3-2t$ ta có bất đẳng thức 
$2t^2+4t^2(3-2t)^2+3 \leq 6t^2+3(3-2t)^2\\  \Leftrightarrow 3t^4-8t^3+3t^2+6t-4 \leq 0 \\ \Leftrightarrow (t-1)^2(3t^2-2t-4) \leq 0$
Hiển nhiên đúng ,vì $2t \leq 3$ nên $3t^2-2t-4 \leq 0$.
Đẳng thức xảy ra khi và chỉ khi $a=b=c=1.\blacksquare$

1.2.Định lí dồn biến S.M.V.

Gỉa sử $a_1,a_2,..,a_n$ là dãy số thực tuỳ ý .Ta thực hiện liên tiếp các biến đổi sau 
1.  Chọn $i,j \in\ {{1,2,...,n}}$ là 2 số thực sao cho 
$a_i=min(a_1,a_2,..,a_n), a_j=max(a_1,a_2,..,a_n).$
2. Thay $a_i \ \text{và} \ a_j \ \text{Bởi} \ \dfrac{a_i+a_j}{2}$(nhưng vẫn giữ đúng thứ tự của chúng  trong dãy số )
Khi đó sau vô hạn lần thực hiện biến đổi nói trên thì mỗi số $a_i$ đề tiến tới giới hạn.
$a=\dfrac{a_1+a_2+..+a+n}{n}$

Ví dụ:   CMR với mọi số thực dương a,b,c,d ta có:
$a^4+b^4+c^4+d^4+2abcd \geq a^2b^2+b^2c^2+c^2d^2+d^2a^2+a^2c^2+b^2d^2$
Giải: Giả sử $a\geq b \geq c \geq d$ .Xét 
$f(a,b,c,d)=a^4+b^4+c^4+d^4+2abcd-a^2b^2-b^2c^2-c^2d^2-d^2a^2-a^2c^2-b^2d^2 \\ =a^4+b^4+c^4+d^4+2abcd-a^2c^2-b^2d^2-(a^2+c^2)(b^2+d^2) \\ \implies f(a,b,c,d)-f(\sqrt{ac},b,\sqrt{ac},d)=(a^2-c^2)^2-(b^2+d^2)(a-c)^2 \geq 0$
Theo S.M.V, xét với phép biến đổi $\Delta$ của $(a,b,c )$ ta chỉ cần chứng mính các bất đẳng thức khi $a=b=c=d=t \geq 0.$
Bất đẳng thức lúc này tương đương với 
$3t^4+d^4+2t^3d \geq 3t^4+3t^2d^2 \Leftrightarrow d^4+t^3d+t^3d \geq 3t^2d^2$.
Hiển nhiên đúng theo AM-GM .Đẳng thức xảy ra khi chỉ khi $a=b=c=d$ hoặc $a=c=b,d=0$ hoặc các hoán vị.
1.3.Phương pháp dồn biến toàn miền
Ví dụ: Chứng minh với mọi số thực không âm a,b,c ta luôn có 
$a^3+b^3+c^3-3abc \geq 4ab(a-b)(b-c)(c-a)$
Giải:   
$BDT\iff (a+b+c)[(a-b)^2+(b-c)^2+(c-a)^2]\ge 8(a-b)(b-c)(c-a) \ (1)$
Không mất tính tổng quát ta giả sử $c=\min (a,b,c)$. Cố định các các hiệu $a-b.b-c.c-a$ và giảm $a,b,c$ cùng 1 lượng $c$( Tức là thay $a,b,c$ bởi $a-c,b-c,0$) thì rõ ràng $a-b,a-c,b-c$ không thay đổi còn $a+b+c$ giảm đi. Vậy VT của $(1)$ thì giảm đi còn VP của (1) thì không đổi. Do đó ta chỉ cần chứng minh Bài toán trong trường hợp $a,b\ge c=0$. Khi đó BĐT tương đương với:
$a^3+b^3\ge 4(ab(b-a)$
BĐT trên hiẻn nhiên đúng vì
$4a(b-a)\le b^2\implies 4 ab(b-a)\le b^3\le a^3+b^3$
Đẳng thức xảy ra $\iff a=b=c. \blacksquare$
  2.Phương pháp phân tích bình phương S.O.S

Xét biểu thức $S=f(a,b,c)=S_a(b-c)^2+S_b(a-c)^2+S_c(a-b)^2$
Trong đó $S_a,S_b,S_c$ là các hàm số của a,b,c.
1.Nếu $S_a,S_b,S_c \geq 0$ thì $S \geq 0$
2.Nếu $a \geq b \geq c \geq 0 \ va \ S_b,S_b+S_c,S_c+S_a \geq 0$ thì $S \geq 0$
3.Nếu $a \geq b \geq c \geq 0 \ va\ S_a,S_c,S_a+2S_b,S_c+2S_a \geq 0$ thì $S \geq 0$
4.Nếu $a \geq b \geq c \geq 0 \ va \ S_b,S_c \geq 0,a^2S_b+b^2S_a \geq 0$ thì $S \geq 0$
5.Nếu $S_a+S_b+S_c \geq 0$ và $S_aS_b+S_bS_c+S_cS_a$ thì $S \geq 0$
Ví dụ :  CMR với các số thực không âm x,y,z ta luôn có bất đẳng thức 
$\dfrac{1}{(x+y)^2}+\dfrac{1}{(y+z)^2}+\dfrac{1}{(z+x)^2} \geq \dfrac{9}{4(xy+yz+zx)}$
Giải :  Đặt $a=x+y,b=y+z,c=z+x.$  Ta phải chứng minh:
$(2ab+2bc+2ca-a^2-b^2-c^2)(\dfrac{1}{a^2}+\dfrac{1}{b^2}+\dfrac{1}{c^2}) \geq \dfrac{9}{4}$
Bằng các phép biến đổi đơn giản ,ta có thể biến các bất đẳng thức trên về dạng: 
$\left(\dfrac{2}{bc}-\dfrac{1}{a^2}\right)(b-c)^2+\left(\dfrac{2}{ca}-\dfrac{1}{b^2}\right)(a-c)^2$ $+\left(\dfrac{2}{ab}-\dfrac{1}{c^2}\right)(a-b)^2 \geq 0$
$\Rightarrow S_a=\dfrac{2}{bc}-\dfrac{1}{a^2},S_b=\dfrac{2}{ca}-\dfrac{1}{b^2},S_c=\dfrac{2}{ab}-\dfrac{1}{c^2}.$
Giả sử rằng $a \geq b \geq c$ thì $S_a \geq 0$ .Sử dụng 4 ta cần CM: 
$b^2S_b+c^2S_c \geq 0\Leftrightarrow b^3+c^3 \geq bc(b+c) \geq abc$
Đẳng thức xảy ra khi chỉ khi $a=b=c$ hoặcc $a=b,c=0$ hoặc hoán vị.

Thứ Hai, 2 tháng 9, 2013

Bất đẳng thức Nesbitt mở rộng.

$$\sum^n_{i=1} \dfrac{a^k_i}{s-a_i}\ge \dfrac{\sum^n_{i=1} a^{k-1}_i}{n-1}$$
Với $s=a_1+a_2+...+a_n; \ a_1,a_2;a_3,....,a_n\ge 0$ và $k\ge 1$

Olympiad 30-4

Bài toán:   Chứng minh rằng với các số thực $a,b,c\in [1,2]$ ta có bất đẳng thức:
$$(a+b+c)\left(\dfrac{1}{a}+\dfrac{1}{b}+\dfrac{1}{c}\right)\le 10$$

 Lời giải:  Ta có BĐT tương đương với:
$$\dfrac{a}{b}+\dfrac{b}{c}+\dfrac{c}{a}+\dfrac{b}{a}+\dfrac{c}{b}+\dfrac{a}{c}\le 7$$
Không mất tính tổng quát ta giải sử $ a\ge b\ge c$. Khi đó:
$$(a-b)(b-c)\ge 0  \iff ab+bc\ge b^2+ac\iff \dfrac{a}{c}+1\ge \dfrac{a}{b}+\dfrac{b}{c} \\ ab+bc\ge b^2+ac\iff \dfrac{c}{a}+1\ge \dfrac{c}{b}+\dfrac{b}{a} $$
Do đó:
$$\dfrac{a}{b}+\dfrac{b}{c}+\dfrac{c}{b}+\dfrac{b}{a}\le \dfrac{a}{c}+\dfrac{c}{a}+2 \\ \implies \dfrac{a}{b}+\dfrac{b}{c}+\dfrac{c}{a}+\dfrac{b}{a}+\dfrac{c}{b}+\dfrac{a}{c}\le 2+2\left(\dfrac{a}{c}+\dfrac{c}{a}\right)$$
Đặt $x=\dfrac{a}{c}$ ta có $1\le x\le 2\implies (x-2)(x-1)\le 0\implies x+\dfrac{1}{x}\le \dfrac{5}{2}$
Từ đó suy ra đpcm. Đẳng thức xảy ra khi $a=b=2,c=1$ hoặc $a=2;b=c=1$ hoặc các hoán vị tương ứng của chúng. $\blacksquare$
Bài toán. Giải phương trình $$\left( {3{x^2} + 2x + 7} \right)\sqrt {2x + 7}  - \left( {3{x^2} - 7x + 26} \right)\sqrt {x - 1}  = 2{x^3} + 14{x^2} - 2x + 22$$

Giải. Điều kiện: $x \ge 1$.

Ta nhẩm được nghiệm $x = 1$nên nghĩ tới trục căn thức, nên thêm vào hai vế của phương trình đại lượng $ - 3\left( {3{x^2} + 2x + 7} \right)$ ta được:

$$\left( {3{x^2} + 2x + 7} \right)\sqrt {2x + 7}  - 3\left( {3{x^2} + 2x + 7} \right) - \left( {3{x^2} - 7x + 26} \right)\sqrt {x - 1}  = 2{x^3} + 14{x^2} - 2x + 22 - 3\left( {3{x^2} + 2x + 7} \right)$$
$$ \Leftrightarrow \left( {3{x^2} + 2x + 7} \right)\left( {\sqrt {2x + 7}  - 3} \right) - \left( {3{x^2} - 7x + 26} \right)\sqrt {x - 1}  = 2{x^3} + 5{x^2} - 8x + 1$$
$$ \Leftrightarrow \frac{{2\left( {x - 1} \right)\left( {3{x^2} + 2x + 7} \right)}}{{\sqrt {2x + 7}  + 3}} - \left( {3{x^2} - 7x + 26} \right)\sqrt {x - 1}  = \left( {x - 1} \right)\left( {2{x^2} + 7x - 1} \right) \\ \iff x=1$$

Do:
$$\dfrac{{2\left( {3{x^2} + 2x + 7} \right)\sqrt {x - 1} }}{{\sqrt {2x + 7}  + 3}} \le \frac{{2\left( {3{x^2} + 2x + 7} \right)\sqrt {x - 1} }}{{3 + 3}} = \left( {{x^2} + \frac{2}{3}x + \frac{7}{3}} \right)\sqrt {x - 1}  \le \left( {2{x^2} + 7x - 1} \right)\sqrt {x - 1} ,\forall x \ge 1$$
và $3{x^2} - 7x + 26 > 0,\forall x \ge 1$. Do đó phương trình vô nghiệm.

Do vậy phương trình có nghiệm duy nhất $x = 1$.
Bài toán:Cho $a+b+c= \dfrac{1}{a}+\dfrac{1}{b}+\dfrac{1}{c}$
chứng minh $(ab+bc+ca)(\sqrt{ab}+\sqrt{bc}+\sqrt{ca})^2\ge 27$ với $a,b,c>0$
Lời giải: Đặt $x=ab \ ; \ y=bc \ ; \ z=ca\Rightarrow a=\sqrt{\dfrac{xz}{y}} \ ; \ b=\sqrt{\dfrac{xy}{z}} \ ; \ c=\sqrt{\dfrac{yz}{x}} \Rightarrow x+y+z=xy+yz+zx$
Bất đẳng thức cần chứng minh tương đương với:
$$(x+y+z)(\sqrt{x}+\sqrt{y}+\sqrt{z})^2 \ge 27 \iff \sqrt{x}+\sqrt{y}+\sqrt{z} \ge \dfrac{3\sqrt{3}}{ \sqrt{x+y+z}}$$
Viết lại bất đẳng thức với dạng:
$$\sqrt{\dfrac{x}{x+y+z}}+\sqrt{\dfrac{y}{x+y+z}}+ \sqrt{ \dfrac{z}{x+y+z}} \ge \dfrac{3\sqrt{3}(xy+yz+zx)}{(x+y+z)^2}$$
$$\iff 2\left(\sqrt{\dfrac{x}{x+y+z}}+\sqrt{\dfrac{y}{x+y+z}}+\sqrt{\dfrac{z}{x+y+z}} \right)+\dfrac{3\sqrt{3}(x^2+y^2+z^2)}{(x+y+z)^2} \ge 3\sqrt{3}$$
Theo Cauchy thì:
$$\dfrac{3\sqrt{3}x^2}{(x+y+z)^2}+\sqrt{\dfrac{x}{x+y+z}}+\sqrt{\dfrac{x}{x+y+z}} \ge \dfrac{3\sqrt{3}x}{x+y+z}$$
Như vậy:
$$VT \ge 3\sqrt{3}\left(\dfrac{x}{x+y+z}+\dfrac{y}{x+y+z}+ \dfrac{ z}{ x+y+z} \right)=3\sqrt{3}=VP$$
Diều phải chứng minh. Đẳng thức xảy ra khi $x=y=z=1\iff a=b=c=1$
Bài toán:    Cho x,y,z là độ dài 3 cạnh của 1 tam giác. CMR:
$(x+y+z)(\dfrac{1}{x}+\dfrac{1}{y}+\dfrac{1}{z})+ \dfrac{3(x-y)(y-z)(z-x)}{xyz} \ge 9$

Lời giải: BĐTcần chứng minh tương đương với :
$$3+\sum \dfrac{x}{y}+\sum \dfrac{y}{x}+3\sum \dfrac{x}{y}-3\sum \dfrac{y}{x}\geq 9 \\ \Leftrightarrow 2\sum \dfrac{x}{y}\geq \sum \dfrac{y}{x}+3$$
Có thể có nhiều cách chứng minh bất đẳng thức trên nhưng có lẽ cách đơn giản nhất là dùng ... $EMV$
Ta viết lại bất đẳng thức cần chứng minh 
$$2(\dfrac{x}{y}+\dfrac{y}{z}+\dfrac{z}{x})\geq 3+(\dfrac{y}{x}+\dfrac{z}{y}+\dfrac{x}{z}) \\ \Leftrightarrow 3(\dfrac{x}{y}+\dfrac{y}{z}+\dfrac{z}{x}-\dfrac{y}{x}-\dfrac{z}{y}-\dfrac{x}{z})+(\dfrac{x}{y}+\dfrac{y}{z}+\dfrac{z}{x}+\dfrac{y}{x}+\dfrac{z}{y}+\dfrac{x}{z}-6)\geq 0 \\ \Leftrightarrow 3(x-y)(y-z)(z-x)+x(y-z)^{2}+y(z-x)^{2}+z(x-y)^{2}\geq 0\  (*)$$ 
Đặt $f(a,b$,$c$)$=VT(*)$.
Theo $EMV$ thì $f(a,b$,$c$)$\geq$ $f(a-x,b-x,c-x)$ với $x$ là một số thực dương thoả mãn $x\leq max${$a$,$b$,$c$} và ($a-x$,$b-x$,$c-x$) là $3$ cạnh của tam giác.
Không mất tính tổng quát, giả sử $a=max${$a$,$b$,$c$}, theo bất đẳng thức tam giác thì :
$$b-x+c-x\geq a-x\Leftrightarrow x\leq b+c-a$$
Suy ra $x\in [0;b+c-a]$
Theo $EMV$, ta chỉ cần chứng minh bất đẳng thức trên với $a=b+c$, tức là :
$$3yz(z-y)+y^{3}+z^{3}+(y+z)(y-z)^{2}\geq 0 \\ \Leftrightarrow 2(y^{3}+z^{3})+2yz^{2}-4y^{2}z\geq 0 \\ \Leftrightarrow 2y(y-z)^{2}+2z^{3}\geq 0$$
Bất đẳng thức cuối hiển nhiên đúng, suy ra đpcm.
Dấu bằng xảy ra khi $a=b=c$

Bài toán: Cho x, y, z là các số thực dương thỏa mãn $x + y + z = xyz$. Chứng minh rằng:
$$\dfrac{1 + \sqrt[]{1 + x^2}}{x} + \dfrac{1 + \sqrt[]{1 + y^2}}{y} + \dfrac{1 + \sqrt[]{1 + z^2}}{z} \le xyz$$
Lời giải: Theo giả thiết tồn tại ba góc nhọn của một tam giác thỏa mãn $x = \tan A;y = \tan B;z = \tan C$.
Ta có $$\dfrac{{1 + \sqrt {1 + {x^2}} }}{x} = \dfrac{{1 + \sqrt {1 + {{\tan }^2}A} }}{{\tan A}} = \dfrac{{1 + \dfrac{1}{{\cos A}}}}{{\dfrac{{\sin A}}{{\cos A}}}} = \dfrac{{\cos A + 1}}{{\sin A}} = c{\rm{ot}}\frac{A}{2}$$
Tương tự ta có
$$\dfrac{{1 + \sqrt {1 + {y^2}} }}{y} = \cot \dfrac{B}{2};\dfrac{{1 + \sqrt {1 + {z^2}} }}{z} = \cot \dfrac{C}{2}$$
Khi đó $$VT = \cot \dfrac{A}{2} + \cot \dfrac{B}{2} + \cot \dfrac{C}{2} \le 3\sqrt 3  \le \tan A.\tan B.\tan C = VP$$.
Bất đẳng thức được chứng minh.

Chủ Nhật, 1 tháng 9, 2013

Bài toán: Cho $a,b,c>0$ thỏa mãn $15\left( {\dfrac{1}{{{a^2}}} + \dfrac{1}{{{b^2}}} + \dfrac{1}{{{c^2}}}} \right) = 10\left( {\dfrac{1}{{ab}} + \dfrac{1}{{bc}} + \dfrac{1}{{ca}}} \right) + 2011$ Tìm giá trị lớn nhất của
$P=\dfrac{1}{\sqrt{5a^2+2ab+2b^2}}+\dfrac{1}{\sqrt{5b^2+2bc+2c^2}}+\dfrac{1}{\sqrt{5c^2+2ac+2a^2}}$
Lời giải:     Đặt $x=\dfrac{1}{a};y=\dfrac{1}{b};z=\dfrac{1}{c}\\  \rightarrow 15\left(x^{2}+y^{2}+z^{2} \right)=10\left(xy+yz+zx \right)+2011\leq 10\left(x^{2}+y^{2}+z^{2} \right)+2011 \\ \rightarrow x^{2}+y^{2}+z^{2}\leq \dfrac{2011}{5}$.
Mặt khác $\left(x+y+z \right)^{2}\leq 3\left(x^{2}+y^{2}+z^{2} \right)\rightarrow \left(x+y+z \right)^{2}\leq 3.\dfrac{2011}{5} $
Ta có $\dfrac{1}{\sqrt{5a^{2}+2ab+b^{2}}}\leq \dfrac{1}{\sqrt{4a^{2}+2ab+b^{2}+2ab}}=\dfrac{1}{2a+b}\leq \dfrac{1}{9}\left(\dfrac{2}{a}+\dfrac{1}{b} \right)=\dfrac{1}{9}\left(2x+y \right)$
Tương tự ta có $P\leq \dfrac{1}{3}\left(x+y+z \right)\leq \dfrac{1}{3}\sqrt{3.\dfrac{2011}{5}}$

Bất đẳng thức

Bài toán: Với mọi $x,y,z>0$.Chứng minh:
$\sqrt {{x^2} + xy + {y^2}}  + \sqrt {{y^2} + yz + {z^2}}  + \sqrt {{z^2} + zx + {x^2}}  \ge \sqrt 3 (x + y + z)$
Lời giải: Sử dụng bất đẳng thức ${{x}^{2}}+{{y}^{2}}\ge \dfrac{{{(x+y)}^{2}}}{2}$, ta có
$$2\sqrt{{{x}^{2}}+xy+{{y}^{2}}}=\sqrt{2{{(x+y)}^{2}}+2({{x}^{2}}+{{y}^{2}})}\ge \sqrt{2{{(x+y)}^{2}}+{{(x+y)}^{2}}}=\sqrt{3}(x+y)$$
Tương tự,
$$2\sqrt{{{y}^{2}}+yz+{{z}^{2}}}\ge \sqrt{3}(y+z) \\ 2\sqrt{{{z}^{2}}+zx+{{x}^{2}}}\ge \sqrt{3}(z+x)$$
Cộng theo vế các bất đẳng thức trên thu được điều cần chứng minh